Keresés

Részletes keresés

NevemTeve Creative Commons License 2011.03.16 0 0 60658

> Egy kérdést tettem fel,

 

Igen, azt, hogy igaz-e, hogy arc sin (cos x) = sqrt (1-x2). Azért kérdezted, mert magadtól nem tudtad, vagy milyen okból?

Előzmény: Scoola (60655)
NevemTeve Creative Commons License 2011.03.16 0 0 60654

Már mondtam, hogy ez így nem megy. Vagy vállald fel, hogy te képviseled itt az ő álláspontját, és akkor megvitathatjuk, vagy kész, vége, zárjuk le. (Off: Egyébként ránéztem, tényleg hülyeség.)

Előzmény: Scoola (60652)
ivivan Creative Commons License 2011.03.16 0 0 60653

"De annyira zárt matematikailag, hogy nem tudtunk belekötni"

 

Matematikailag? Nincs is benne matek :-) Csak dobálózik a szavakkal, mintha keverné a fizikát és a filozófiát.

 

Az a kényszerképzete, hogy a specrel valami látszat, nem a valóság. Szerintem a GPS órák járása épp elég bizonyíték arra, hogy a relativitás elmélet valóságos és nem csak mérési hiba. És persze ezen kívül van még tucatnyi olyan mérési eredmény, amely a látszólagossággal nem magyarázható.

Előzmény: Scoola (60651)
ivivan Creative Commons License 2011.03.16 0 0 60650

"Látatlanban miért gondolod, hogy te találnál benne?"

 

Miért gondolod, hogy Gézoo itt nem írta még le az okosságait? Túl vagyunk már rajta...

Előzmény: Scoola (60648)
NevemTeve Creative Commons License 2011.03.16 0 0 60649

Akkor ezt lezárthatjuk, várjuk a követekező cáfolót.

Előzmény: Scoola (60648)
ivivan Creative Commons License 2011.03.16 0 0 60647

háháááá, Gézoo :-)

 

Már annyira hiányzik... Persze nem írja le, hogy miért is lenne ez attól csak "mérési hiba", mert a képletet átrendezi...

Előzmény: Scoola (60645)
NevemTeve Creative Commons License 2011.03.16 0 0 60646

ad 1: ez nem így működik; ha te akarsz valamiről beszélgetni, akkor te add elő a saját szavaiddal, úgy, ahogy megértetted; egyébként hagyjuk süllyedni.

 

ad 2: egy fizikai elméletben, amelyik igazából nem más, mint tiszta matematika, nincs helye mérési hibának... ez valami süvöltő marhaság lesz

Előzmény: Scoola (60645)
NevemTeve Creative Commons License 2011.03.16 0 0 60644

Valószínű, hogy az a valaki, aki valahol valamit írt, tévedett, vagy félreértett valamit. Ha tudsz konkértabbat mondani, akkor bővebben is megbeszélhetjük.

 

Előzmény: Scoola (60642)
Scoola Creative Commons License 2011.03.16 0 0 60642

Egy szkeptikus oldalon olvastam a "tudjuk kinek" a levezetését, abban ez a ß=1/sin(arccos(v/c)) függvény volt a kérdéses. A levezetés többi része vitathatatlanul korrekt. Ő nevezte mérési hibának a mérési hibát.

Előzmény: NevemTeve (60640)
ivivan Creative Commons License 2011.03.16 0 0 60641

Elárulnád, hogy miért csak egy mérési hiba attól, mert a képletet átrendezzük?

Előzmény: Scoola (60636)
NevemTeve Creative Commons License 2011.03.16 0 0 60640

Azt viszont elárulhatnád, hogy milyen mérést végeztél, és milyen hibát tapasztaltál?

Előzmény: Scoola (60639)
Scoola Creative Commons License 2011.03.16 0 0 60639

Rendben. Úgy is hit alapú a relativitás, miért ne hihetnék a becsületszavadban is?

Előzmény: NevemTeve (60638)
NevemTeve Creative Commons License 2011.03.16 0 0 60638

Nem akarok most 'számítástudomány alapelemei' órát tartani, kérlek hidd el becsületszóra.

Előzmény: Scoola (60637)
Scoola Creative Commons License 2011.03.16 0 0 60637

Három művelet gyorsabb mint két művelet? Milyen értelemben gyorsabb?

Előzmény: NevemTeve (60634)
Scoola Creative Commons License 2011.03.16 0 0 60636

Kösz! Tehát igaz. Csak egy mérési hiba. Megdöbbentő.

Előzmény: egy mutáns (60635)
egy mutáns Creative Commons License 2011.03.16 0 0 60635
Előzmény: Scoola (60633)
NevemTeve Creative Commons License 2011.03.16 0 0 60634

Mert a gyökvonás és a négyzetre-emelés gyorsabb, mint a cosinus meg az arcus-sinus.

Előzmény: Scoola (60633)
Scoola Creative Commons License 2011.03.16 0 0 60633

A specrelben nem hibának nevezzük. Rendben. A kérdés az volt, hogy a ß=1/sin(arccos(v/c)) függvény adhat-e azonos értékeket a gamma függvényével? Ha igen, akkor miért nem ezt a függvényt használjuk a transzformációinkban?

Előzmény: egy mutáns (60632)
egy mutáns Creative Commons License 2011.03.16 0 0 60632

Nem mérési hibákról van szó a specrelben. 

1m

Előzmény: Scoola (60629)
egy mutáns Creative Commons License 2011.03.16 0 0 60631

Így van, köszönöm a kiegészítést!

1m

Előzmény: ivivan (60630)
ivivan Creative Commons License 2011.03.16 0 0 60630

Gyorsan átfutva nem találtam benne hibát, de:

 

"A vonat rendszerében pont a kjözépső utasnál, a sín rendszerében valahol a baktertől a vonat menetirányába nézve valahol elől"

 

ez a mondat szerintem félreérthető, bár kétségtelenül igaz. Szóval a sín rendszerében is a középső utasnál találkozik a két fényjel, ami persze a "baktertől a vonat menetirányába nézve valahol elől" van.

Előzmény: egy mutáns (60628)
Scoola Creative Commons License 2011.03.16 0 0 60629

Csak azért kérdezem, mert akkor ha ß=1/sin(arccos(v/c)) valóban azonos értékeket képez a gammával, akkor a vonat hossza nem változik, "csupán rövidebbnek látszik a gamma arányú mérési hiba következtében".

  Ez a feltételezés igaz lenne?

Előzmény: Scoola (60627)
egy mutáns Creative Commons License 2011.03.16 0 0 60628

Kedves Bnum, tekintsük a következő példát, és kérlek, írd róla meg a véleményedet.

A sínen megy egy vonat, üvegből van belelátunk. A vonat két vége tükrös, a fényt visszaveri. 

A vonat közepén van egy utas, nála egy lámpa, de nincs benne elem. 

A sín mellett áll egy bakter, nála van egy elem. 

Amikor a középeső utas odaér a bakter mellé, a bakter benyújtja az elemet, és felvillan a lámpa. 

Tehát igazak a következők:

A lámpa felvillanásakor a fény pontosan a vonat közepéről indul a vonat eleje és vége felé, mégpedig mind a sín, mind a vonat rendszerében, egy időpillanatban, legyen ez a nulla időpillanat mindkét rendszerben, t=0, t'=0, ahol t a vonat rendszerében múló idő, t' pedig a sín rendszerében múló idő, mindegy is most, hogy milyen viszonyban vannak egymással. Jelölje továbbá x a vonaton az utastól mért távolságot (előre pozitív), x' pedig a sínen a bakterttől mért távolságot (előre pozitív), mindegy is most, hogy milyen viszonyban vannak egymáshoz képest.  

 

Mikor ér a 2 fénysugár a vonat elején és végén levő tükrökhöz?

 

1. A vonat rendszerében

t=L/(2c) idő múlva, egy időben, ahol L a vonat hossza a vonat rendszerében. 

 

2. A sín rendszerében

legyen a vonat hossza L', mindegy is most, hogy mennyi L-hez képest. 

A vonat eleje a t'=0 pillanatban L'/2 helyen, a vége pedig -L'/2 helyen van a bakterhez képest. 

Egy valamely későbbi t' pillanatban pedig x'e=L'/2+v*t' helyen van az eleje, a vége pedig x'v=-L'/2+v*t' 

Mikor ér az előrefele tartó fény a vonat elejéhez? A t'e pillanatban?

t'e*c=L'/2+v*t'e, t'e=L'/2/(c-v)

Mikor ér a hátrafele tartó fény a vonat végéhez? A t'v pillanatban?

-t'v*c=-L'/2+v*t'v, t'v=L'/2/(c+v)

 

Látható, hogy t'e=/=t'v

 

Vagyis az a két esemény, hogy a (mind a sín, mind a bakter rendszerében a vonat közepéről induló) 2 fényjel a megérkezik a tükrökhöz, egyidejű a vonat rendszerében, és nem egyidejű a sín rendszerében. 

 

A példa folytatható azzal, hogy mikor és hol találkozik a két visszavert fény. 

A vonat rendszerében pont a kjözépső utasnál, a sín rendszerében valahol a baktertől a vonat menetirányába nézve valahol elől. 

 

A példát legjobb tudásom szerínt írtam le. Van-e benne valami, amit eltévesztettem, vagy érthetetlenül írtam meg?

Beszéljünk róla. 

1m

Előzmény: BBnum (60623)
Scoola Creative Commons License 2011.03.16 0 0 60627

  Érdekes, állítólag a ß=1/sin(arccos(v/c)) függvény azonos értékeket ad a gammával.

Lehetséges lenne?

egy mutáns Creative Commons License 2011.03.16 0 0 60626

érdemesebb volna "rövidebb"-et írni a "megrövidül" helyett.

Így van.

1m

Előzmény: Törölt nick (60550)
ivivan Creative Commons License 2011.03.16 0 0 60625

"Az utólsó állításoddal te is ezt állítottad."

 

Ajjaj. Nem ekvivalens az az állítás, hogy a két elektromos jel egyszerre ér a vonat közepére minden IR szerint (ez triviális, hiszen különben mire lehetne használni a specrelt?) és az, hogy a fényjel egyszerre éri el a baktert.

Előzmény: BBnum (60623)
ivivan Creative Commons License 2011.03.16 0 0 60624

"Ebből következően az érvelésetek hibás."

 

Miért is lenne az? Mert te toppantottál? Öten magyarázzuk neked, mégse megy a fejedbe!

 

Igen, az órák egyetlen rendszer szerint lesznek szinkronban, amelyik szerint állnak. Az összes többi mozgó IR-ben az órák nem lesznek szinkronban. Ezt mondjuk már jópár napja! Ezt próbáld megérteni, elfogadni, megemészteni!

 

Miután elfogadtad (és csak utána!) lehet gondolkozni az ilyen bombás paradoxonokon. Ezek érdekesek, és még a specrel logikáját is meg lehet tanulni belőlük. De jelenleg még semmit nem értesz, különben nem írnál ilyen marhaságokat...

Előzmény: BBnum (60622)
BBnum Creative Commons License 2011.03.16 0 0 60623

"A bakter ezt mondja: elindul előre és hátra fénysebességgel a fényjel. A vonat vége szembe jön, így hamar eléri a fény. A vonat eleje fut a fény elöl, így azt később éri el a fény.

Az elektromos jel a vonat végéből c sebességgel korábban indul el, mint a vonat elejéről. De most meg hátsó elektromos jel kergeti a vonat közepét, míg a vonat elejéről jövő elektromos jellel szembe fut a vonat közepe. Így pont egyszerre érnek, amennyivel később ért az első fényjel, annyival hamarabb ér az elektromos. Szépen egyszerre érnek be és nem robban a vonat."

 

OK. Az érvelés kitünő. Tehát bár a szinkronizálási jelek eltérő időben érnek a vonat végeihez, a visszafelé tartó jelek középre pont egyszerre érnek.

 

Mit is állítok már hosszú oldalakon keresztül?

1) a vonat rendszerében szinkronizált órák vannak, így az ottani megfigyelőt egyszerre érik a jelek.

 

2) a baktert a vonat végéből (vagy az azzal szinkronizált) jelek egyszerre érik el.

 

Az utólsó állításoddal te is ezt állítottad.

A két vég közötti eltérést csak a bakter észleli, de a végekről jövő jel pontosan kikompenzálja az eltérést és ezért a középre (vagy a közép vonalba) vissza érkező jelek egyszerre érnek célba.

Előzmény: mmormota (60621)
BBnum Creative Commons License 2011.03.16 0 0 60622

Akor az eredetit, hogy ne legyen vita:

"a hátulsó óra előbb kapja meg az indító impulzust, mint az elülső. Ugyanis a hátulsó óra a felé küldött impulzussal szemben halad, míg az elülső óra a felé küldött impulzus irányával egyező irányban halad - mintha menekülne a felé küldött impulzus elől. Menekülésének csak annyi eredménye van, hogy ő később kapja meg az indítóimpulzusát, mint a hátulsó óra a sajátját."

 

Tehát a szinkronizálási eljárás során keletkezik az órák eltérése, utánna már együtemben járnak, de az eltérés (hiába szinkronizáltunk) fent marad.

 

Az eltérés megfigyelő függő.

 

Erre hoztam a példámat a másik vonattal.

 

De mégegyszer: a szinkronizálás során egyáltalán nem biztos, hogy létezik másik megfigyelő, illetve később több eltérő sebességű külső megfigyelő is lehet.

Ebből következően az érvelésetek hibás.

Az órák eltérése (ha létezik) semmiképpen se a szinkronizálással függhet össze, csak jól szinkronizált órák hibás megfigyelésével.

 

"nem egyszerre indulnak el, minden eltérő sebességű megfigyelő számára más és más lesz,"

Ez is jó módszer, az órák beszélgetnek: "Téged figyelnek már? Még nem. Akkor el ne indúlj!"

 

Mivel feltehetőleg az óra nem engem és egyébb megfigyelőket figyel, hanem én figyelem az órát, ezért nem reagálhat az én megfigyelésemre.

A szinkronizált órák jeleit a sebesség miatt (?) eltérőnek észlelem, de valójában az órák szinkronba járnak.

Csak az érvelés itt is sántit, mert mind két óra sebessége azonos.

Hiába figyelnénk Galilei hajóját, a legyek és szunyogok eloszlása nem változna, hiába mozognék én v sebességgel a hajóhoz képest.

Előzmény: mmormota (60620)
mmormota Creative Commons License 2011.03.16 0 0 60621

"A bakter úgy látja felrobbant a vonat? :O)"

 

Nem látja úgy. Az hogy robban vagy nem robban, minden megfigyelő számára egyforma kell legyen, különben nyugodtan el lehetne dobni a specrelt.

 

A vonatos megfigyelő is, a bakter is úgy látja hogy nem robban fel, de a részletekben már más lesz a mese.

 

A vonatos azt mondja: előre és hátra c sebességgel ment a fény, egyszerre érte el a vonat elejét és végét, onnan fénysebességgel jött vissza az elektromos jel, egyszerre ért középre, így nincs robbanás.

 

A bakter ezt mondja: elindul előre és hátra fénysebességgel a fényjel. A vonat vége szembe jön, így hamar eléri a fény. A vonat eleje fut a fény elöl, így azt később éri el a fény.

Az elektromos jel a vonat végéből c sebességgel korábban indul el, mint a vonat elejéről. De most meg hátsó elektromos jel kergeti a vonat közepét, míg a vonat elejéről jövő elektromos jellel szembe fut a vonat közepe. Így pont egyszerre érnek, amennyivel később ért az első fényjel, annyival hamarabb ér az elektromos. Szépen egyszerre érnek be és nem robban a vonat.


(ha az elektromos jel nem c sebességgel hanem akármekkorával halad akkor is ugyanez az eredmény, csak akkor számolgatni is kell a meséhez)

Előzmény: BBnum (60618)

Ha kedveled azért, ha nem azért nyomj egy lájkot a Fórumért!